LSAT and Law School Admissions Forum

Get expert LSAT preparation and law school admissions advice from PowerScore Test Preparation.

 Administrator
PowerScore Staff
  • PowerScore Staff
  • Posts: 8917
  • Joined: Feb 02, 2011
|
#26299
Complete Question Explanation

Resolve the Paradox. The correct answer choice is (A)

The paradox in this stimulus involves what is known as a “counterproductive strategy.” A counterproductive strategy exacerbates the problem it is intended to solve. Here, highways are being widened and extended in order to reduce traffic congestion and delays, yet this strategy only makes the problem worse. The correct answer choice should offer a possible explanation as to why this strategy backfires.

Answer Choice (A): This is the correct answer choice. If the widened and extended roads attract more motorists, then this could cause increased congestion and delays. Many correct answers on Resolve the Paradox questions provide a possible cause for the paradox in the stimulus. This answer choice is no exception.

Answer Choice (B): This answer choice may explain why the road widening and extension projects are undertaken in the first place, but it does not explain why this strategy increases delays and congestion.

Answer Choice (C): Even if this answer choice is true, it fails to explain the discrepancy between the intended result of the highway project and the actual result.

Answer Choice (D): The paradox in the stimulus is only about highways near urban areas. This answer choice concerns rural traffic. This answer may explain congestion on rural roads, but it does not explain why the above strategy is counterproductive on urban roads.

Answer Choice (E): This answer would resolve a paradox similar to the paradox presented in the stimulus. This might explain why urban traffic is more congested and delayed initially. But this answer choice does not explain why extending and widening roads causes an increase, rather than a decrease, in congestion and delays.
User avatar
 PresidentLSAT
  • Posts: 87
  • Joined: Apr 19, 2021
|
#87137
Hello,

I have a question: is it reasonable to rule out B because an increase in population doesn't necessarily infer an increase in motorists?
 Robert Carroll
PowerScore Staff
  • PowerScore Staff
  • Posts: 1787
  • Joined: Dec 06, 2013
|
#87224
President,

That's a problem with answer choice (B). It also doesn't explain why congestion got worse after the project if the project would only have been undertaken after the population increase.

Robert Carroll

Get the most out of your LSAT Prep Plus subscription.

Analyze and track your performance with our Testing and Analytics Package.